Đến nội dung

KietLW9 nội dung

Có 1000 mục bởi KietLW9 (Tìm giới hạn từ 24-05-2020)



Sắp theo                Sắp xếp  

#731989 $1\leq \frac{x}{1-yz}+\frac{y...

Đã gửi bởi KietLW9 on 12-12-2021 - 16:23 trong Bất đẳng thức và cực trị

Cách khác: Áp dụng bất đẳng thức AM - GM ta có $\sum \frac{x}{1-yz}=\sum\frac{x}{x^2+y^2+z^2-yz}\leq \sum\frac{2x}{2x^2+y^2+z^2}=\sum\frac{2x}{x^2+\frac{1}{3}+\frac{1}{3}+\frac{1}{3}}\leq \sum\frac{2x}{4\sqrt[4]{x^2.\frac{1}{27}}}=\frac{\sqrt[4]{27}}{2}\sum\sqrt{x}\leq \frac{\sqrt[4]{27}}{2}\sqrt{3(x+y+z)}\leq \frac{3\sqrt{3}}{2}$.

Còn $\sum\frac{x}{1-yz}\geq x+y+z\geq \sqrt{x^2+y^2+z^2}=1$.

P/s: Cách ông Kiệt đoạn Min nhân thêm $x,y,z$ vô mẫu nên hình như phải xét riêng trường hợp.

Là sao nhỉ?  :(




#731982 $1\leq \frac{x}{1-yz}+\frac{y...

Đã gửi bởi KietLW9 on 11-12-2021 - 20:09 trong Bất đẳng thức và cực trị

$\because \frac{x}{1-yz}+\frac{y}{1-zx}+\frac{z}{1-xy}\geqslant 1$

Áp dụng Cauchy-Schwarz, ta được: $\frac{x}{1-yz}+\frac{y}{1-zx}+\frac{z}{1-xy}=\frac{x^2}{x-xyz}+\frac{y^2}{y-xyz}+\frac{z^2}{z-xyz}\geqslant \frac{(x+y+z)^2}{(x+y+z)-3xyz}\geqslant x+y+z\geqslant \sqrt{x^2+y^2+z^2}=1$ (Vì $x,y,z$ không âm)

Đẳng thức xảy ra khi trong 3 số $x,y,z$ có 2 số bằng 0 và 1 số bằng 1.

$\because \frac{x}{1-yz}+\frac{y}{1-zx}+\frac{z}{1-xy}\leqslant \frac{3\sqrt{3}}{2}$

Ta có: $\frac{x}{1-yz}\leqslant \frac{x}{1-\frac{y^2+z^2}{2}}=\frac{2x}{1+x^2}\leqslant \frac{3\sqrt{3}}{8}x^2+\frac{3\sqrt{3}}{8} \text{ (UCT)}$

Tương tự rồi cộng lại, ta được $\frac{x}{1-yz}+\frac{y}{1-zx}+\frac{z}{1-xy}\leqslant \frac{3\sqrt{3}}{2}$

Đẳng thức xảy ra khi $x=y=z=\frac{1}{\sqrt{3}}$




#726208 $0\leq a\leq b \leq c\leq 1$; P=$(a+b+c).(...

Đã gửi bởi KietLW9 on 29-04-2021 - 22:09 trong Bất đẳng thức và cực trị

Đây thực chất là bài toán: Cho $1\leqslant x\leqslant y\leqslant z\leqslant 2$. Tìm GTLN của: $P=(x+y+z)(\frac{1}{x}+\frac{1}{y}+\frac{1}{z})$




#726827 $\textrm{Cho } x,y,z \textrm{ thỏa mãn }...

Đã gửi bởi KietLW9 on 10-05-2021 - 21:11 trong Bất đẳng thức và cực trị

$\textrm{Cho } x,y,z \textrm{ thỏa mãn } x+y+z=0;x+1>0;y+1>0 \textrm{ và } z+4>0.$

$\textrm{Tìm GTLN của } A=\dfrac{xy-1}{(x+1)(y+1)}+\dfrac{z}{z+4}$

Đặt $x+1=a;y+1=b;z+4=c$ thì $a,b,c>0$ và $a+b+c=6$

Khi đó $A=\frac{(a-1)(b-1)-1}{ab}+\frac{c-4}{c}=\frac{ab-a-b}{ab}+\frac{c-4}{c}=2-(\frac{1}{a}+\frac{1}{b}+\frac{4}{c})\leqslant 2-\frac{(1+1+2)^2}{a+b+c}=\frac{-2}{3}$

Đẳng thức xảy ra khi $x=y=\frac{1}{2};z=-1$




#727186 $\textrm{Cho } x,y \textrm{ là các số thực khôn...

Đã gửi bởi KietLW9 on 17-05-2021 - 20:15 trong Bất đẳng thức và cực trị

Tại của em nó gọn quá ng` ta đou có hỉu

Anh không hiểu ý của em á? Ý của em là như này: Đúng là bài của em gọn thật nhưng em muốn ghi thế thành dãy cho nó đẹp, còn bài của anh thì dòng đầu đã có $P\leqslant \frac{1}{4}$ thì anh suy ra giá trị lớn nhất của $\text{P}$ được luôn chứ sao lại ghi thêm $\frac{1}{4}-P\geqslant 0$ (vì sau khi chứng minh điều này là hiển nhiên). Vả lại dòng $2,3,4$ của anh nó gọn y hệt của em nữa?




#727161 $\textrm{Cho } x,y \textrm{ là các số thực khôn...

Đã gửi bởi KietLW9 on 17-05-2021 - 09:25 trong Bất đẳng thức và cực trị

$\textrm{Lời giải được trình bày như sau:}$

$\textrm{Ta có:}$

$P \le \dfrac{(x^2-0)(1-x^2.0)}{(1+x^2)^2(1+0)^2}=\dfrac{x^2}{(1+x^2)^2} \le \dfrac{x^2}{4x^2} =\dfrac14$

$\Rightarrow \dfrac14 - P \ge 0$

$\Leftrightarrow \dfrac14-\dfrac{(x^2-y^2)(1-x^2y^2)}{(1+x^2)^2(1+y^2)^2} \ge 0$

$\Leftrightarrow \dfrac{(y^4+1)(x^2-1)^2+y^2(6x^4+4x^2+6)}{4(1+x^2)^2(1+y^2)^2} \ge 0 \textrm{(luôn đúng)}$

$\textrm{Dấu "=" xảy ra} \Leftrightarrow \left \{ {{x=\pm 1} \atop {y=0}} \right.$

$\textrm{Vậy ...}$

$\text{Khúc này có gì khác của em đâu anh? Với lại từ dòng đầu là hình như đã xong bài rồi mà?}$ :mellow:




#727113 $\textrm{Cho } x,y \textrm{ là các số thực khôn...

Đã gửi bởi KietLW9 on 16-05-2021 - 09:20 trong Bất đẳng thức và cực trị

Lời giải. Ta có: 

$\frac{1}{4}-\dfrac{(x^2-y^2)(1-x^2y^2)}{(1+x^2)^2(1+y^2)^2}=\frac{(y^4+1)(x^2-1)^2+y^2(6x^4+4x^2+6)}{4(1+x^2)^2(1+y^2)^2}\geqslant 0$

Đẳng thức xảy ra khi $(x,y)\in\left \{ (1,0);(-1,0) \right \}$




#729930 $\sum_{cyc}^{}\frac{1}{a^3(...

Đã gửi bởi KietLW9 on 27-08-2021 - 09:06 trong Bất đẳng thức - Cực trị

Đặt $(\frac{1}{a},\frac{1}{b},\frac{1}{c})\rightarrow (x,y,z)\Rightarrow \left\{\begin{matrix}x,y,z>0 & \\ xyz=1 & \end{matrix}\right.$

Khi đó bất đẳng thức được viết lại thành: $\frac{x^2}{y+z}+\frac{y^2}{z+x}+\frac{z^2}{x+y}\geqslant \frac{3}{2}$

Không mất tính tổng quát, giả sử $x\geqslant y\geqslant z\geqslant 0\Rightarrow \left\{\begin{matrix}x+y\geqslant z+x\geqslant y+z>0 & \\ \frac{x}{y+z}\geqslant \frac{y}{z+x}\geqslant \frac{z}{x+y}>0 & \end{matrix}\right.$

Sử dụng bất đẳng thức Chebyshev cho hai dãy đơn điệu cùng chiều và ngược chiều, ta được: $\frac{x^2}{y+z}+\frac{y^2}{z+x}+\frac{z^2}{x+y}\geqslant \frac{1}{3}(x+y+z)(\frac{x}{y+z}+\frac{y}{z+x}+\frac{z}{x+y})=\frac{1}{6}[(y+z)+(z+x)+(x+y)](\frac{x}{y+z}+\frac{y}{z+x}+\frac{z}{x+y})\geqslant \frac{1}{6}.3.[\frac{x}{y+z}.(y+z)+\frac{y}{z+x}.(z+x)+\frac{z}{x+y}.(x+y)]=\frac{x+y+z}{2}\geqslant \frac{3\sqrt[3]{xyz}}{2}=\frac{3}{2}$
Đẳng thức xảy ra khi $a=b=c=1$



#725504 $\sum_{cyc} \frac{1}{a(a+b)}...

Đã gửi bởi KietLW9 on 16-04-2021 - 18:15 trong Bất đẳng thức và cực trị

chứng minh đi bạn :)

Troll bạn tí thôi chứ thật ra bài toán mình đưa ra thực chất là bài toán của bạn. Bằng cách chuẩn hóa $abc = 1$ và đặt như bạn là ra :D




#725538 $\sum_{cyc} \frac{1}{a(a+b)}...

Đã gửi bởi KietLW9 on 17-04-2021 - 09:13 trong Bất đẳng thức và cực trị

Quên nói với bạn

Cái này ko chuẩn hóa được :))

https://diendantoanh...c32sqrt3a2b2c2/




#725484 $\sum_{cyc} \frac{1}{a(a+b)}...

Đã gửi bởi KietLW9 on 16-04-2021 - 16:56 trong Bất đẳng thức và cực trị

Ta chứng minh được với mọi $a,b,c>0$ thì ta luôn có: $\frac{1}{a(a+b)}+\frac{1}{b(b+c)}+\frac{1}{c(c+a)}\geq \frac{3}{2\sqrt[3]{a^{2}b^{2}c^{2}}}$




#729784 $\sum_{a}^{c}\frac{a^{3}...

Đã gửi bởi KietLW9 on 17-08-2021 - 15:01 trong Bất đẳng thức - Cực trị

$\sum \frac{a^3}{b+c}=\sum \frac{a^4}{ab+ac}\geqslant \frac{(a^2+b^2+c^2)^2}{2(ab+bc+ca)}\geqslant \frac{(a^2+b^2+c^2)^2}{2(a^2+b^2+c^2)}=\frac{a^2+b^2+c^2}{2}$




#729799 $\sum_{a}^{c}\frac{a^{2}(a-...

Đã gửi bởi KietLW9 on 18-08-2021 - 10:27 trong Bất đẳng thức - Cực trị

Ta cần chứng minh: $\frac{a^3}{a+b}+\frac{b^3}{b+c}+\frac{c^3}{c+a}\geqslant \frac{a^2b}{a+b}+\frac{b^2c}{b+c}+\frac{c^2a}{c+a}$

Dễ thấy:

+) $\frac{a^3}{a+b}+\frac{b^3}{b+c}+\frac{c^3}{c+a}=\frac{a^4}{a^2+ab}+\frac{b^4}{b^2+bc}+\frac{c^4}{c^2+ca}\geqslant \frac{(a^2+b^2+c^2)^2}{a^2+b^2+c^2+ab+bc+ca}\geqslant \frac{(a^2+b^2+c^2)^2}{a^2+b^2+c^2+a^2+b^2+c^2}=\frac{a^2+b^2+c^2}{2}$

+) $\frac{a^2b}{a+b}+\frac{b^2c}{b+c}+\frac{c^2a}{c+a}\leqslant \frac{a^2b}{2\sqrt{ab}}+\frac{b^2c}{2\sqrt{bc}}+\frac{c^2a}{2\sqrt{ca}}=\frac{\sqrt{a^3b}+\sqrt{b^3c}+\sqrt{c^3a}}{2}\leqslant \frac{\sqrt{3(a^3b+b^3c+c^3a)}}{2}\leqslant \frac{a^2+b^2+c^2}{2}$




#732160 $\sum{\frac{1}{1+a+b}}\leq...

Đã gửi bởi KietLW9 on 20-12-2021 - 16:53 trong Bất đẳng thức và cực trị

Xét hiệu: $\frac{2}{a+2}-\frac{b}{ab+b+1}-\frac{1}{a+b+1}=\frac{a(b-1)^2}{(a+2)(ab+b+1)(a+b+1)}\geqslant 0\Rightarrow \frac{2}{a+2}\geqslant \frac{b}{ab+b+1}+\frac{1}{a+b+1}$

Tương tự rồi cộng lại, ta có: $\frac{2}{a+2}+\frac{2}{b+2}+\frac{2}{c+2}\geqslant \frac{b}{ab+b+1}+\frac{c}{bc+b+1}+\frac{a}{ca+c+1}+\frac{1}{a+b+1}+\frac{1}{b+c+1}+\frac{1}{c+a+1}=\frac{1}{a+b+1}+\frac{1}{b+c+1}+\frac{1}{c+a+1}+1$

Đến đây cần chứng minh: $\frac{1}{a+2}+\frac{1}{b+2}+\frac{1}{c+2}\leqslant 1\Leftrightarrow \frac{a}{a+2}+\frac{b}{b+2}+\frac{c}{c+2}\geqslant 1$

Thật vậy: $\frac{a}{a+2}+\frac{b}{b+2}+\frac{c}{c+2}=\frac{a}{a+2\sqrt[3]{abc}}+\frac{b}{b+2\sqrt[3]{abc}}+\frac{c}{c+2\sqrt[3]{abc}}=\sum_{cyc}^{}\frac{\sqrt[3]{a^2}}{\sqrt[3]{a^2}+2\sqrt[3]{bc}}\geqslant \sum_{cyc}^{}\frac{\sqrt[3]{a^2}}{\sqrt[3]{a^2}+\sqrt[3]{b^2}+\sqrt[3]{c^2}}=1$




#726017 $\sum\sqrt{7a+9}\ge 10$

Đã gửi bởi KietLW9 on 27-04-2021 - 15:58 trong Bất đẳng thức và cực trị

Cho $a,b,c\ge 0$ thỏa mãn $a+b+c=1$. Chứng minh rằng $$\sqrt{7a+9}+\sqrt{7b+9}+\sqrt{7c+9}\ge 10.$$

Đề thi HK1 Toán 9 - Huyện Đan Phượng HN - 2017 - 2018.

Xét bất đẳng thức phụ: $\sqrt{7a+9}\geqslant a+3\Leftrightarrow 7a+9\geqslant a^2+6a+9\Leftrightarrow a(a-1)\leqslant 0$*đúng*

Tương tự rồi cộng lại: $\sqrt{7a+9}+\sqrt{7b+9}+\sqrt{7c+9}\geqslant a+b+c+9=10$

Đẳng thức xảy ra khi trong 3 số $a,b,c$ có 2 số bằng 0 và 1 số bằng 1




#725994 $\sum\sqrt{\frac{b+c}{a}}...

Đã gửi bởi KietLW9 on 26-04-2021 - 14:46 trong Bất đẳng thức và cực trị

Chứng minh : $\sqrt{\frac{b+c}{a}}+\sqrt{\frac{a+c}{b}}+\sqrt{\frac{a+b}{c}}\geq 2(\sqrt{\frac{a}{b+c}}+\sqrt{\frac{b}{c+a}}+\sqrt{\frac{c}{a+b}})$ với mọi $a,b,c> 0$

Bất đẳng thức cần chứng minh tương đương: $(\sqrt{\frac{a+b}{c}}-\frac{2\sqrt{c}}{\sqrt{a+b}})+(\sqrt{\frac{b+c}{a}}-\frac{2\sqrt{a}}{\sqrt{b+c}})+(\sqrt{\frac{c+a}{b}}-\frac{2\sqrt{b}}{\sqrt{c+a}})\geqslant 0\Leftrightarrow \frac{b+c-2a}{\sqrt{a(b+c)}}+\frac{c+a-2b}{\sqrt{b(c+a)}}+\frac{a+b-2c}{\sqrt{c(a+b)}}\geqslant 0$

Giả sử $a\geqslant b\geqslant c$ thì $b+c-2a\leqslant c+a-2b\leqslant a+b-2c$ và $\frac{1}{\sqrt{a(b+c)}}\leqslant \frac{1}{\sqrt{b(c+a)}}\leqslant \frac{1}{\sqrt{c(a+b)}}$

Sử dụng bất đẳng thức Chebyshev cho 2 dãy đơn điệu cùng chiều $b+c-2a\leqslant c+a-2b\leqslant a+b-2c$ và $\frac{1}{\sqrt{a(b+c)}}\leqslant \frac{1}{\sqrt{b(c+a)}}\leqslant \frac{1}{\sqrt{c(a+b)}}$, ta được: $3(\frac{b+c-2a}{\sqrt{a(b+c)}}+\frac{c+a-2b}{\sqrt{b(c+a)}}+\frac{a+b-2c}{\sqrt{c(a+b)}})\geqslant (b+c-2c+c+a-2b+a+b-2c)(\frac{1}{\sqrt{a(b+c)}}+ \frac{1}{\sqrt{b(c+a)}}+ \frac{1}{\sqrt{c(a+b)}})=0$

Vậy ta có điều phải chứng minh

Đẳng thức xảy ra khi $a = b = c$ 




#727461 $\sum\frac{x + y}{xy + z^{2}}\leq \sum\frac{1}{...

Đã gửi bởi KietLW9 on 22-05-2021 - 20:48 trong Bất đẳng thức và cực trị

Bất đẳng thức sai với $\lceil (x,y,z)\rightarrow (-1,-2,-3)\rfloor$




#726908 $\sum\frac{x^3}{x^2+y^2} \geq \f...

Đã gửi bởi KietLW9 on 12-05-2021 - 21:16 trong Bất đẳng thức và cực trị

Cho $x,y,z$ là các số thực dương. Chứng minh rằng:

$$\frac{x^3}{x^2+y^2}+\frac{y^3}{y^2+z^2}+\frac{z^3}{z^2+x^2} \geq \frac{x+y+z}{2}$$

Ta có: $\frac{x^3}{x^2+y^2}-\frac{2x-y}{2}=\frac{y(x-y)^2}{2(x^2+y^2)}\geqslant 0$

Tương tự rồi cộng lại, ta có điều phải chứng minh




#727054 $\sum\frac{a^4}{b^4+c^4}\geq\sum...

Đã gửi bởi KietLW9 on 15-05-2021 - 09:52 trong Bất đẳng thức và cực trị

Cho ba số dương a, b, c CMR:

$\frac{a^4}{b^4+c^4}+\frac{b^4}{a^4+c^4}+\frac{c^4}{a^4+b^4}\geq \frac{a^2}{b^2+c^2}+\frac{b^2}{c^2+a^2}+\frac{c^2}{a^2+b^2}$

 

$VT-VP=(a^4+b^4+c^4+a^2b^2+b^2c^2+c^2a^2)\sum_{cyc}\frac{b^2c^2(b+c)^2(b-c)^2}{(c^4+a^4)(a^4+b^4)(c^2+a^2)(a^2+b^2)}$




#725134 $\sum\frac{a^{2}}{b}\geq...

Đã gửi bởi KietLW9 on 09-04-2021 - 12:05 trong Bất đẳng thức - Cực trị

Ta dễ có: $\sum_{cyc}\sqrt{a^2-ab+b^2}=\sum_{cyc}\sqrt{\frac{1}{4}(a+b)^2+\frac{3}{4}(a-b)^2}\geqslant \sum_{cyc}\sqrt{\frac{1}{4}(a+b)^2}=a+b+c$

Suy ra $\sum_{cyc}\frac{a^2}{b}=\sum_{cyc}(\frac{a^2-ab+b^2}{b}+b)-(a+b+c)\geqslant \sum_{cyc}2\sqrt{a^2-ab+b^2}-(a+b+c)= \sum_{cyc}\sqrt{a^2-ab+b^2}+\sum_{cyc}\sqrt{a^2-ab+b^2}-(a+b+c)\geqslant \sum_{cyc}\sqrt{a^2-ab+b^2}$ 

Đẳng thức xảy ra khi a = b = c




#726530 $\sum x^2+xyz=4 \to \sum x\le3$

Đã gửi bởi KietLW9 on 06-05-2021 - 11:07 trong Bất đẳng thức và cực trị

Cho các số thực dương $x,y,x$ thoả $x^2+y^2+z^2+xyz=4.$ Chứng minh

$$ x+y+z\le 3.$$

Vì $x^2+y^2+z^2+xyz=4$ nên tồn tại các số $a,b,c$ sao cho $\left\{\begin{matrix}x=2\sqrt{\frac{bc}{(a+b)(a+c)}} & \\ y=2\sqrt{\frac{ca}{(b+a)(b+c)}} & \\ z=2\sqrt{\frac{ab}{(c+a)(c+b)}} & \end{matrix}\right.$

Ta cần chứng minh: $2\sqrt{\frac{bc}{(a+b)(a+c)}}+2\sqrt{\frac{ca}{(b+a)(b+c)}}+2\sqrt{\frac{ab}{(c+a)(c+b)}}\leqslant 3$

Thật vậy, theo bất đẳng thức Cô-si, ta có: $2\sqrt{\frac{bc}{(a+b)(a+c)}}+2\sqrt{\frac{ca}{(b+a)(b+c)}}+2\sqrt{\frac{ab}{(c+a)(c+b)}}\leqslant 3\leqslant (\frac{b}{a+b}+\frac{c}{a+c})+(\frac{c}{b+c}+\frac{a}{a+b})+(\frac{a}{c+a}+\frac{b}{c+b})=3(Q.E.D)$

Đẳng thức xảy ra khi $a=b=c$ hay $x=y=z=1$



#727095 $\sum cyc\frac{1}{(a+b)^2+4abc}+\frac{a^2+b^2+c^2}{8}...

Đã gửi bởi KietLW9 on 15-05-2021 - 16:51 trong Bất đẳng thức và cực trị

a,b,c>0.CM: $\sum cyc\frac{1}{(a+b)^2+4abc}+\frac{a^2+b^2+c^2}{8}\geq \sum cyc\frac{1}{a+3}$

Lời giải. Bất đẳng thức cần chứng minh có thể được viết về dạng: $(\frac{8}{(a+b)^2+4abc}+\frac{a^2+b^2}{2})+(\frac{8}{(b+c)^2+4abc}+\frac{b^2+c^2}{2})+(\frac{8}{(c+a)^2+4abc}+\frac{c^2+a^2}{2})\geqslant \frac{8}{a+3}+\frac{8}{b+3}+\frac{8}{c+3}$

Áp dụng bất đẳng thức Cauchy, ta được: $\frac{8}{(a+b)^2+4abc}+\frac{a^2+b^2}{2}\geqslant \frac{8}{(a+b)^2+c(a+b)^2}+\frac{(a+b)^2}{4}=\frac{8}{(c+1)(a+b)^2}+\frac{(a+b)^2}{4}\geqslant 2\sqrt{\frac{8}{(c+1)(a+b)^2}.\frac{(a+b)^2}{4}}=\frac{8}{2\sqrt{2(c+1)}}\geqslant \frac{8}{c+3}$

Tương tự rồi cộng lại, ta có điều phải chứng minh

Đẳng thức xảy ra khi $a=b=c=1$




#727237 $\sum bc\sqrt{a^{2}-1}\leq \frac...

Đã gửi bởi KietLW9 on 18-05-2021 - 13:53 trong Bất đẳng thức - Cực trị

Lời giải. Bài này có thể giải bằng lượng giác hóa




#725440 $\sum a^3 +3abc \geq \sum ab(a+b)$

Đã gửi bởi KietLW9 on 15-04-2021 - 17:08 trong Bất đẳng thức và cực trị

Cho các số thực $a,b,c \geq 0 $.Chứng minh rằng:

$a^3 + b^3 + c^3 +3abc \geq ab(a+b)+bc(b+c)+ca(c+a)$

Đây là bất đẳng thức Schur dạng $a^k(a-b)(a-c)+b^k(b-a)(b-c)+c^k(c-a)(c-b)\geqslant 0$ với k = 1 :lol:

Chứng minh: 

Giả sử $a\geqslant b\geqslant c\geqslant 0$ thì $c(c-a)(c-b)\geqslant 0\Leftrightarrow c^3+abc\geqslant ac^2+bc^2\Leftrightarrow a^3+b^3+c^3+3abc\geqslant ac^2+bc^2+2abc+a^3+b^3$

Ta cần có: $ac^2+bc^2+2abc+a^3+b^3\geqslant ab(a+b)+bc(b+c)+ca(c+a)\Leftrightarrow (a+b-c)(a-b)^2\geqslant 0$*đúng*

Đẳng thức xảy ra khi a = b = c hoặc a = b, c = 0 và các hoán vị




#724862 $\sum {{{\left( {a + b} \right)...

Đã gửi bởi KietLW9 on 03-04-2021 - 12:07 trong Bất đẳng thức và cực trị

Việt Nam TST 1996 | Let solution say the method